Show that for any two integers a, b , (a+b)^2 ≡ a^2 + b^2 (mod 2)

  • Thread starter Thread starter Simkate
  • Start date Start date
  • Tags Tags
    Integers
Click For Summary
The discussion focuses on proving that for any two integers a and b, (a+b)^2 is congruent to a^2 + b^2 modulo 2. The proof demonstrates that since 2x = 0 (mod 2) for any integer x, the term 2ab becomes 0 in the equation (a+b)^2 = a^2 + 2ab + b^2. This leads to the conclusion that (a+b)^2 ≡ a^2 + b^2 (mod 2). An alternative method mentioned is proving by exhaustion, but the general proof provided is sufficient and applicable to any prime. The proof is confirmed to be correct and well-structured.
Simkate
Messages
24
Reaction score
0
Show that for any two integers a, b , (a+b)^2 ≡ a^2 + b^2 (mod 2)

I have my solution below i wanted someone to help chekc if i have done anything wrong. Thank You for your help.

The thing that is going on here is that 2x = 0 (mod 2) for any x. If x = ab, then 2ab = 0 (mod 2).
We see that (a+b)^2 = a^2 + 2ab + b^2. Then this equals a^2 + 0 + b^2 (mod 2).

Therfore, the Proof:

By definition, showing (a+b)^2 = a^2 + b^2 (mod 2) is equivalent to showing that

2 divides [(a+b)^2 - (a^2+b^2)]
2 divides [(a^2 + 2ab + b^2 - a^2 - b^2]
2 divides 2ab

To show 2 divides 2ab we need to find an integer k such that 2k = 2ab. Take k = ab. Thus it is proved.
 
Physics news on Phys.org
This looks like a good proof.


However... you already know modular arithmetic, right?
We see that (a+b)^2 = a^2 + 2ab + b^2. Then this equals a^2 + 0 + b^2 (mod 2).
Then this is a complete proof.



(For the record, you could have also proven it by exhaustion -- there are only four cases, and they're easy to check by hand. However, the argument you used generalizes to replacing 2 by any prime)
 
Question: A clock's minute hand has length 4 and its hour hand has length 3. What is the distance between the tips at the moment when it is increasing most rapidly?(Putnam Exam Question) Answer: Making assumption that both the hands moves at constant angular velocities, the answer is ## \sqrt{7} .## But don't you think this assumption is somewhat doubtful and wrong?

Similar threads

  • · Replies 5 ·
Replies
5
Views
2K
  • · Replies 1 ·
Replies
1
Views
1K
  • · Replies 9 ·
Replies
9
Views
2K
  • · Replies 4 ·
Replies
4
Views
2K
  • · Replies 3 ·
Replies
3
Views
896
Replies
15
Views
4K
  • · Replies 16 ·
Replies
16
Views
3K
Replies
7
Views
4K
Replies
14
Views
4K
Replies
2
Views
1K